14
$\begingroup$

Let   $p_1\ p_2\ \ldots$ be the sequence of all natural prime numbers. There is a slight (just slight) but clear tendency for imitating the number of primes in an interval $(p_k;\ p_n)$   by the number of primes in the double interval   $(p_k\!+p_{k+1};\ p_{n-1}\!+p_n)$; possibly by   $(2\cdot p_k; 2\cdot p_n)$   too. Let me ask two open questions along this line. The first one will be most likely hopeless while the second one may lead to a discussion and at least to numerical computations.

P1.   Does there exist a natural number   $d$   such that for every natural number   $n$   the real interval

$$ (2\cdot p_n;\ 2\cdot p_{n+d})$$

contains at least one prime?

P2.   (when P1 fails): Given a natural number   $d$,   let   $w(d)$   be the least natural number such that the interval of P1 (see above) does not contain any prime number. What is the growth of the sequence

$$w(1)\ \ w(2)\ \ w(3)\ \ldots$$


The above notions got shifted from my original definition by a half of a prime. The question Q1 below is still equivalent to question P1 above:

Q1.   Does there exist a natural number   $d>1$   such that for every natural number   $n$   the real interval

$$ (p_n\!+p_{n+1};\ p_{n+d-1}\!+p_{n+d})$$

contains at least one prime?

Q2.   (when Q1 fails): Given a natural number   $d>1$,   let   $v(d)$   be the least natural number such that the interval of Q1 (see above) does not contain any prime number. What is the growth of the sequence

$$v(1)\ \ v(2)\ \ v(3)\ \ldots$$

EXAMPLE   Consider the consecutive primes

$$p_{360} = 1901 \qquad p_{361}=1907 \qquad p_{362}=1913$$

Then the real interval

$$(p_{360}\!+p_{361};\ p_{361}\!+p_{362})\ \ =\ \ (3802; 3820)$$

contains no primes, i.e.   $v(2)\le 360$.


In general, I'd be interested in similar relative properties of primes, where primes are studied in relations to other primes, and the relation is not trivial, meaning not reduced to general properties between integers.

$\endgroup$
6
  • 1
    $\begingroup$ You might consider the growth of maximal prime gaps. I expect v exists and is superexponential. See Tomas Silva and Nicely web pages on gaps to start. $\endgroup$ Sep 3, 2013 at 5:44
  • $\begingroup$ @TMA, I'll get hold of the paper. Thank you. $\endgroup$ Sep 3, 2013 at 18:27
  • 1
    $\begingroup$ I also read of a conjecture which asserts a pair of twin primes between $p_n^2$ and $p_{n+1}^2$. $\endgroup$ Sep 3, 2013 at 23:20
  • 1
    $\begingroup$ Perhaps. Not as ambitious as the (failed) conjecture that there are a pair of twin primes between $n^2$ and $(n+1)^2$. $\endgroup$ Sep 3, 2013 at 23:55
  • 1
    $\begingroup$ @TheMaskedAvenger : Ask for a prime only between two consecutive squares ( still unsolved if I am right). $\endgroup$ Jul 24, 2016 at 23:53

2 Answers 2

23
$\begingroup$

The answer to P1 is negative, thanks to the recent work of Maynard on bounded gaps between primes.

What Maynard shows is that given any $d$, there exists a k-tuple $h_1,\dots,h_k$ such that for infinitely many $n$, at least $d+1$ of $n+h_1,\dots,n+h_k$ are prime. In fact, the argument shows that for sufficiently large $x$, the number of $n \in [x,2x]$ such that at least $m$ of $n+h_1,\dots,n+h_k$ are prime, and the rest are almost prime in the sense that they have no prime factor less than $x^\varepsilon$ for some small fixed $\varepsilon>0$, is $\gg \frac{x}{\log^k x}$. (Such strengthenings of Zhang/Maynard type theorems are discussed in this paper of Pintz, and also in this Polymath8b preprint.) A standard upper bound sieve (e.g. Selberg sieve or beta sieve) then shows that after removing about $O( x/\log^{k+1} x)$ of these $n$, one can also ensure that none of the numbers between $2(n+h_1)$ and $2(n+h_k)$ are prime. If we let $p_i$ be the first prime greater than or equal to $n+h_1$, then we have $2(n+h_1) \leq 2p_i \leq 2p_{i+d} \leq 2(n+h_k)$, and so we obtain a counterexample to P1 for any $d$.

Unfortunately we don't get an effective rate on P2 this way due to the reliance on the Bombieri-Vinogradov theorem in the work of Maynard etc. However this can likely be removed by following the ideas mentioned near the end of this paper of Pintz, though I did not attempt this. [EDIT: it looks likely that the quantitative version of Maynard's results in this recent paper of Banks, Freiberg, and Maynard will do the trick, after some small modification.]

$\endgroup$
17
$\begingroup$

The answer to your first question is most surely no. Certainly this follows from the Hardy-Littlewood $k$-tuple conjectures. Choose $n$ so that the primes $p_n$, $\ldots$, $p_{n+d}$ are all very close to each other. Hardy-Littlewood says that there are lots of such tuples. Then you are asking for one more prime in the very short interval $[2p_n,2p_{n+d}]$ (essentially) and the sieve could be used to show that this happens rarely. This will also give a conjectural way of tackling question 2 -- just find a small admissible $d$-tuple (which has been discussed before on MO).

It may even be possible to prove this unconditionally, although I don't fully see this. By Westzynthius/Erdos-Rankin you can construct largish intervals without primes, and if you halve that interval usually you will get a reasonable number of primes there.

$\endgroup$
0

Your Answer

By clicking “Post Your Answer”, you agree to our terms of service and acknowledge you have read our privacy policy.

Not the answer you're looking for? Browse other questions tagged or ask your own question.